LSAT and Law School Admissions Forum

Get expert LSAT preparation and law school admissions advice from PowerScore Test Preparation.

User avatar
 Dave Killoran
PowerScore Staff
  • PowerScore Staff
  • Posts: 5852
  • Joined: Mar 25, 2011
|
#45450
Complete Question Explanation
(The complete setup for this game can be found here: lsat/viewtopic.php?t=2450)

The correct answer choice is (A)

If G drives exactly twice, then the four drivers must be in the 2-2-1-1 distribution, and consequently answer choice (A) is correct.
 deck1134
  • Posts: 160
  • Joined: Jun 11, 2018
|
#49850
Why could we not have

G H/F J G H/F G
M T W T F S

Isn't that in line with the rules? And against (A)?
 Vaidehi Joshi
PowerScore Staff
  • PowerScore Staff
  • Posts: 23
  • Joined: Aug 16, 2018
|
#49854
@deck1134

Remember the question, which says "If during one week Gina drives on Monday and Saturday only"--this means that G drives no other days besides M and S, so it can't be on T, as you have in your hypothetical.
 deck1134
  • Posts: 160
  • Joined: Jun 11, 2018
|
#49877
Oof. Rookie mistake. Thanks

Get the most out of your LSAT Prep Plus subscription.

Analyze and track your performance with our Testing and Analytics Package.